Curl and Cauchy-Riemann Conditions problem

  • Thread starter Thread starter Saketh
  • Start date Start date
  • Tags Tags
    Conditions Curl
Click For Summary
The discussion revolves around verifying the Cauchy-Riemann conditions for a two-dimensional flow of an incompressible and irrotational liquid. The user successfully demonstrated one of the conditions, \(\frac{\partial u}{\partial y} = -\frac{\partial v}{\partial x}\), but struggled with the other condition, \(\frac{\partial u}{\partial x} = \frac{\partial v}{\partial y}\). A key insight provided was that the divergence of the velocity field, \(\nabla \cdot \textbf{V} = 0\), is crucial for proving the second condition, stemming from the continuity equation for incompressible fluids. The user initially misunderstood the implications of incompressibility but gained clarity on its mathematical significance. This exchange highlights the importance of understanding fluid dynamics principles in verifying mathematical conditions.
Saketh
Messages
258
Reaction score
2
Problem

The velocity of a two-dimensional flow of liquid is given by

<br /> \textbf{V} = \textbf{i}u(x, y) - \textbf{j}v(x, y).<br />​

If the liquid is incompressible and the flow is irrotational show that

<br /> \frac{\partial u}{\partial x} = \frac{\partial v}{\partial y}<br />​

and

<br /> \frac{\partial u}{\partial y} = -\frac{\partial v}{\partial x}<br />​

My Work

I evaluated \nabla \times \textbf{V} = 0 through a determinant, and ended up with this expression:

\textbf{i}\frac{\partial v}{\partial z} + \textbf{i}\frac{\partial u}{\partial z} - \textbf{k}\left ( \frac{\partial u}{\partial y} + \frac{\partial v}{\partial z} \right ) = 0<br />​

Through this, I was able to verify:

<br /> \frac{\partial u}{\partial y} = -\frac{\partial{v}}{\partial x}<br />​

I could not verify the other expression. How can I verify the other expression - I've tried everything I can think of. It seems simple, but I am missing something.

Thanks in advance.
 
Last edited:
Physics news on Phys.org
Hello Saketh,

to get

<br /> \frac{\partial u}{\partial x} = \frac{\partial v}{\partial y}<br />

you need to use the fact that div V = 0 for incrompressible liquids.

This follows from the continuity equation:

<br /> \frac{\partial \rho}{\partial t} + \nabla(\rho\,\textbf{V})= 0<br />

and using the fact that the liquid is incompressible, hence the mass density \rho=const.

Regards,

nazzard

P.S.: There's a small typo in your current solution: one \frac command is missing
 
Last edited:
Oh, I thought that incompressible just meant that the fluid flow is behaving ideally, and had no mathematical significance.

Now that you tell me \nabla \cdot \textbf{V} = 0, the answer is obvious. Thanks!
 
Question: A clock's minute hand has length 4 and its hour hand has length 3. What is the distance between the tips at the moment when it is increasing most rapidly?(Putnam Exam Question) Answer: Making assumption that both the hands moves at constant angular velocities, the answer is ## \sqrt{7} .## But don't you think this assumption is somewhat doubtful and wrong?

Similar threads

  • · Replies 7 ·
Replies
7
Views
2K
Replies
4
Views
1K
Replies
2
Views
1K
  • · Replies 19 ·
Replies
19
Views
2K
Replies
3
Views
2K
Replies
10
Views
3K
  • · Replies 1 ·
Replies
1
Views
1K
  • · Replies 8 ·
Replies
8
Views
2K
Replies
21
Views
2K
  • · Replies 4 ·
Replies
4
Views
1K